2. Tutorium, am 18.10.2013

Forumsregeln
Wenn Du Lösungsansätze zu Beispielen suchst oder schreibst, stelle nach Möglichkeit auch die dazugehörenden Angaben zur Verfügung - am besten als Dateianhang, da die meisten Übungsangaben auf Institutshomepages nach einem Semester gelöscht werden.
So haben auch die nächsten Semester noch etwas davon ;)
onki_fronz
Beiträge: 15
Registriert: 27.10.2011, 17:50

Re: 2. Tutorium, am 18.10.2013

Beitrag von onki_fronz »

@max_gain


wieso muss ich bei 3ii eigentlich das integral lösen? - sollte es nicht reichen einfach die Zeitahängigkeit auf die Lösung von 3i zu multiplizieren also ich mein ob nicht schon die zweite Zeile bei deiner berechnng von 3ii ausreichen würde??

sumpe
Beiträge: 252
Registriert: 19.04.2009, 20:07

Re: 2. Tutorium, am 18.10.2013

Beitrag von sumpe »

Max_gain hat geschrieben:Hier mal meine Lösungen!
Bei Bsp 4 iii) fehlt die Interpretation über die Unschärferelation, weis nicht genau was da zu tun ist!
Lg
hast du da bei 3b) bei den integral 1 und 2 wo du es allgemein machst, gleich in der ersten zeile, nicht einen fehler? sin^2 ist doch 1-cos^2 oder machst du da was anderes?

Max_gain
Beiträge: 157
Registriert: 29.08.2011, 09:46

Re: 2. Tutorium, am 18.10.2013

Beitrag von Max_gain »

onki_fronz hat geschrieben:@max_gain


wieso muss ich bei 3ii eigentlich das integral lösen? - sollte es nicht reichen einfach die Zeitahängigkeit auf die Lösung von 3i zu multiplizieren also ich mein ob nicht schon die zweite Zeile bei deiner berechnng von 3ii ausreichen würde??
Ja die zweite Zeile ist bereits eine superposition aus ebenen Wellen!
Es kam mit nur so wenig vor!
sumpe hat geschrieben:
Max_gain hat geschrieben:Hier mal meine Lösungen!
Bei Bsp 4 iii) fehlt die Interpretation über die Unschärferelation, weis nicht genau was da zu tun ist!
Lg
hast du da bei 3b) bei den integral 1 und 2 wo du es allgemein machst, gleich in der ersten zeile, nicht einen fehler? sin^2 ist doch 1-cos^2 oder machst du da was anderes?
Ich verwende die gleiche Formel wie beim dritten Integral nur das x=y ist im ersten fall!

bananenneutrino
Beiträge: 98
Registriert: 10.09.2011, 15:59

Re: 2. Tutorium, am 18.10.2013

Beitrag von bananenneutrino »

Max_gain hat geschrieben:Danke für die vielen hilfreichen Tipps! Aber hättest du vielleicht noch einen zur Berechnung des Erwartungswertes des Impules ohne ein Integral durchführen zu müssen? :)
Naja, den kannst du aus der Impulsverteilung ablesen. Also wenn du die Wellenfunktion des Impulses hast, absolutquadrat gebildet und du hast eine Normalverteilung, die um Null zentriert ist. Also ist der Erwartungswert 0. Genauso kannst du daraus die Standardabweichung ablesen.

bananenneutrino
Beiträge: 98
Registriert: 10.09.2011, 15:59

Re: 2. Tutorium, am 18.10.2013

Beitrag von bananenneutrino »

bananenneutrino hat geschrieben:Die Interpretation bei iii ist ganz einfach, dass die Heisenbergsche Unschärferelation bei t=0 genau erfüllt ist, und danach der Abstand zur Schranke immer größer wird. Dabei gilt, je geringer die Ortsunschärfe am Anfang, desto stärker wächst die Ortsunschärfe mit der Zeit.
Ich bin es noch einmal durchgegangen: Die Heisenbergsche Unschärferelation bleibt für alle Zeiten scharf! Die Impulsunschärfe nimmt mit der Zeit ab, die Ortsunschärfe mit der Zeit zu.

Max_gain
Beiträge: 157
Registriert: 29.08.2011, 09:46

Re: 2. Tutorium, am 18.10.2013

Beitrag von Max_gain »

bananenneutrino hat geschrieben:
bananenneutrino hat geschrieben:Die Interpretation bei iii ist ganz einfach, dass die Heisenbergsche Unschärferelation bei t=0 genau erfüllt ist, und danach der Abstand zur Schranke immer größer wird. Dabei gilt, je geringer die Ortsunschärfe am Anfang, desto stärker wächst die Ortsunschärfe mit der Zeit.
Ich bin es noch einmal durchgegangen: Die Heisenbergsche Unschärferelation bleibt für alle Zeiten scharf! Die Impulsunschärfe nimmt mit der Zeit ab, die Ortsunschärfe mit der Zeit zu.
Da beziehst du dich aber auf den 2ten fall, wo t als Parameter in der standardabweichung ist. Oder?

bananenneutrino
Beiträge: 98
Registriert: 10.09.2011, 15:59

Re: 2. Tutorium, am 18.10.2013

Beitrag von bananenneutrino »

Jap, tu ich.

Vitronius
Beiträge: 44
Registriert: 15.11.2012, 15:10

Re: 2. Tutorium, am 18.10.2013

Beitrag von Vitronius »

Hallo,

wie löst man denn das Integral der e-Funktion in 4b (i)? Also e^(a(x+b/a))^2?
Auf den schönen Wurzelausdruck wie Max Gain komm ich nicht.
Wie löst man das auf?
Danke schon mal im Voraus!

Max_gain
Beiträge: 157
Registriert: 29.08.2011, 09:46

Re: 2. Tutorium, am 18.10.2013

Beitrag von Max_gain »

Das ist das gausintegral!
\int e^{-a(x-b)^{2}}=\sqrt(\frac{\pi}{a})

Kupuspita
Beiträge: 17
Registriert: 11.12.2011, 22:15

Re: 2. Tutorium, am 18.10.2013

Beitrag von Kupuspita »

@Max_gain: bei 3b hast du zum Schluss 2*I_3 abgezogen statt nur I_3. -> <x>=0.41*L

Zu 3c)
Der Erwartungswert des Impulses ist das Skalarprodukt < \varphi , \frac{h_q}{i}\bigtriangledown \varphi)> (Herleitung im Skript (2.36)). Nachdem im linken Teil eine Superposition von ungeraden Funktionen (Sin) ist und im rechten Teil gerade Funktionen sind (Cos) ist das Skalarprodukt (mit der Skalarproduktdefinition in diesem Hilbertraum) demnach 0 (Orthogonalität). ist ein kurzer Einzeiler ohne irgendwelche Integrale zu lösen.
Zuletzt geändert von Kupuspita am 17.10.2013, 22:49, insgesamt 1-mal geändert.
“Physics is like sex: sure, it may give some practical results, but that's not why we do it.” Richard P. Feynman

Vitronius
Beiträge: 44
Registriert: 15.11.2012, 15:10

Re: 2. Tutorium, am 18.10.2013

Beitrag von Vitronius »

@ Max_Gain: Danke herzlich, hab ich übersehen!

@Kupuspita: Das stimmt schon so mit dem 2er, das ist ja beim Ausmultiplizieren eine binomische Formel. Wenn du dein Integral halt so definierst, dass du den 2er reinziehst, dann ziehst dus nur einmal ab. Max_Gain hat ihn aber nicht reingezogen.

bananenneutrino
Beiträge: 98
Registriert: 10.09.2011, 15:59

Re: 2. Tutorium, am 18.10.2013

Beitrag von bananenneutrino »

Kupuspita hat geschrieben:@Max_gain: bei 3b hast du zum Schluss 2*I_3 abgezogen statt nur I_3. -> <x>=0.41*L

Zu 3c)
Der Erwartungswert des Impulses ist das Skalarprodukt < \varphi , \frac{h_q}{i}\bigtriangledown \varphi)> (Herleitung im Skript (2.36)). Nachdem im linken Teil eine Superposition von ungeraden Funktionen (Sin) ist und im rechten Teil gerade Funktionen sind (Cos) ist das Skalarprodukt (mit der Skalarproduktdefinition in diesem Hilbertraum) demnach 0 (Orthogonalität). ist ein kurzer Einzeiler ohne irgendwelche Integrale zu lösen.
Das stimmt so nicht. Wenn du die Terme im Skalarprodukt ausmultiplizierst, wirst du auf 2 Integrale stoßen, die beide nicht Null (!) sind, aber je das Komplex Konjugierte voneinander sind. Da du durch den Impuls ein i bekommst, das Integral selbst aber rein reell ist, fallen sie weg.


Die Eigenfunktionen des Hamiltonians bilden immer eine ONB, die Ableitungen (die Cos Funktion) sind aber nicht Teil der ONB hier. Das Skalarprodukt, dass du hier hast ist Integration von 0 bis L, weswegen es bei cos(Pi*x/L)*sin(2Pix/L) Reibereien gibt. Siehe http://www.wolframalpha.com/input/?i=In ... 2CPi%7D%5D

sebastian
Beiträge: 47
Registriert: 13.11.2011, 17:31

Re: 2. Tutorium, am 18.10.2013

Beitrag von sebastian »

Hier die Musterlösung.
Du hast keine ausreichende Berechtigung, um die Dateianhänge dieses Beitrags anzusehen.

Antworten

Zurück zu „Quantentheorie I“